Questions tagged [sobolev-spaces]

For questions about or related to Sobolev spaces, which are function spaces equipped with a norm that controls both a function and its weak derivatives in some Lebesgue space.

Filter by
Sorted by
Tagged with
2 votes
1 answer
106 views

How to interpret $|\nabla u|$ when using distributions?

In Sobolev spaces, to define the respective norm, we need to be able to interpret $||\nabla u||_{L^2(\Omega)}=\left( \int_{\Omega} |\nabla u|^2 \ dt\right)^{1/2}$ However, how do we interpret $|\...
An old man in the sea.'s user avatar
0 votes
1 answer
119 views

The norm of a distribution (generalized function) in a Sobolev Spaces involving time.

Being a distribution a linear continuous functional in the test function space $\mathcal{D}(0,t)$ (for the purpose of my question), and $$||u ||_{L^p(0,T;V)}:= \left(\int^T_0||u||^p_V \ dt\right)^{1/p}...
An old man in the sea.'s user avatar
2 votes
0 answers
74 views

Existence of trace operator in$ W^{s,p} (D)$

Let $D$ be a smooth, bounded domain. $W^{s,p} (D)$ denotes the fractional Sobolev space defined as in Hitchhiker's Guide to Fractional Sobolev Space. Assume that $s>\frac{1}{p}$. I want to show ...
Kaushik's user avatar
  • 491
0 votes
1 answer
33 views

Deducing this inequality for $u\in H^1(a,b)$

In the following picture, $v,u \in H^1(a,b)$ with $(a,b)\subset \mathbb{R}$. Well, I think there is a typo. How do we get $$u(x)+\int^y_x u'(s)ds\leq u(x) +\sqrt{b-a} ||u'||_{L^2(a,b)}\ ?$$ I'm ...
An old man in the sea.'s user avatar
1 vote
0 answers
35 views

Proof of an inequality involving $Q\in H^{1}(\mathbb{R}^d).$

I am trying to understand a special case of Diamagnetic inequality mentioned in Appendix [B] of the book Nonlinear Dispersive Equations by Tao. Here is the statement, Let $Q\in H_x^{1}(\mathbb{R}^d)...
Student's user avatar
  • 9,196
1 vote
0 answers
81 views

Reference request: definition of Sobolev space on manifolds using positive elliptic operator

I've been reading a paper "Schatten classes on compact manifolds: Kernel conditions" by Delgado and Ruzhansky (https://arxiv.org/abs/1403.6158). In the paper, the authors define the Sobolev space $H^\...
S.Lim's user avatar
  • 531
1 vote
0 answers
103 views

$\Delta u=\sin(u)$ implies $u$ smooth for weak solution $u\in H^1_0(\Omega)$

Let $\Omega\subset\mathbb{R}^n$ be a bounded domain, $\partial\Omega\in C^\infty$. Let $\Delta$ be the standard Laplacian. Suppose $u\in H^1_0(\Omega)$ is a weak solution to $$\begin{cases}\Delta u=\...
Yuxiao Xie's user avatar
  • 8,556
5 votes
1 answer
2k views

Understanding a Sobolev Embedding Theorem

In my adv. Analysis course, we have studied the following Sobolev Embedding Theorem: Let $m\in\mathbb{N}$ and $s>m+d/2$. Then $$H^s(\mathbb{R}^d)\hookrightarrow C_0^m(\mathbb{R}^d)$$That is: $H^...
Lorenzo's user avatar
  • 2,601
1 vote
1 answer
89 views

Equivalence of norms in $L_p$ spaces.

Let $(f,g) \in {W_0}^{1,p}(0,1) \times {L^p}(0,1), p\in[1,\infty]$. I want to prove the equivalence of the two norms $${\left\| {{{{f_x} - g} \over 2}} \right\|_p} + {\left\| {{{{f_x} + g} \over 2}} \...
Gustave's user avatar
  • 1,523
11 votes
2 answers
6k views

How to think of homogeneous Sobolev spaces

The homogeneous Sobolev space $\dot H^s(\mathbb{R}^d)$ can be defined as the completion of $\mathscr{S}(\mathbb{R}^d)$ (the space of Schwartz functions) under the norm $$ \|f\|_{\dot H^s(\mathbb{R}^d)...
Chris's user avatar
  • 5,106
1 vote
1 answer
76 views

Liouville's theorem for conformal mappings: differentiable functions?

Apparently Liouville's theorem for conformal mappings holds for functions in $W^{1,n}$. Am I to understand that potentially, there are everywhere differentiable functions to which it doesn't apply?
Seub's user avatar
  • 6,043
1 vote
1 answer
225 views

Does this inclusion for Besov function spaces hold and how to prove it?

Consider the Besov spaces $B_{p,q} ^s$ consisting of those functions $f$ such that $$f \in W^{n,p} (\mathbb{R}), \quad \int_{0} ^{\infty} \Big | \frac{\omega_p ^2(f^{(n)}, t)}{t^{a}} \Big |^q \frac{...
Marc_Adrien's user avatar
2 votes
1 answer
449 views

Hardy-Littlewood-Sobolev inequality using generalised Young inequality in Lorenz spaces

I want to prove that $$\left| \left| \frac{1}{|x|^a} \ast f \right| \right|_q \lesssim ||f||_p$$ with $1 < p < q < \infty$ and $a= n \left(1 + \frac{1}{q}- \frac{1}{p} \right)$ using a ...
tommy1996q's user avatar
  • 3,354
0 votes
1 answer
101 views

Is a $W^{1,\infty}$ function a continuous function?

Let consider a function $f\in W^{1,\infty}([a,b];\mathbb{R}^n)$. Somebody can suggest me a reference where I could confirm if $f$ is a continuous function, due to $f \in W^{1,\infty}([a,b];\mathbb{R}^...
Ana's user avatar
  • 63
5 votes
0 answers
327 views

Equivalent norms on Sobolev spaces. Only function and last derivative needed to define the norm.

Equip $W_n^{p}[0,1]$ with the norm $$\left\|f\right\|_{W_n^{p}} = \sum_{k=0}^{n}\left\|f^{(k)}\right\|_{L^p}.$$ I want to prove that this norm is equivalent to the norm $$\left\|f\right\|_2 = \left\|...
OgvRubin's user avatar
  • 1,381
0 votes
0 answers
65 views

How to find a constant such that this inequality holds in $H^1$

Let $a$, $b \in \mathbb{R}$. Does there exist a $0 < c \in \mathbb{R}$ such that $c(u(a)^2+u(b)^2) \leq \left\lVert u \right\rVert_{H^1}^2$ for all $u \in H^1(a,b)$? I can't find an explicit bound ...
sessame's user avatar
  • 33
0 votes
1 answer
462 views

Convergence of Lipschitz functions in $L^\infty$ implies the convergence in $H^{1}$

Suppose $f_n \to f$ in $L^\infty(U)$ where $f_n, f \in C^{1,1}(\bar{U})$, (Holder space with $k=1, \alpha =1$, i.e, Lipschitz functions) where $U$ is an open and bounded in $\mathbb{R}^n$. Q: Can we ...
induction601's user avatar
  • 1,944
1 vote
1 answer
56 views

Show $u \notin H^2$ for $-\text{div}(a\Delta u)=1 $ where $a$ is a step function

I am working on a problem which shows interior regularity may fail if you have non-Lipschitz coefficients. Let $\Omega = [−1,1]$ and $K = [−1/2,1/2] \in \Omega$. Define $a(x)=1+1_K(x)$ and ...
user30523's user avatar
  • 1,681
1 vote
0 answers
375 views

Weak solution of Laplace equation in $H^2$

Let $\Omega = [0,1]$ and $f\in L^2(\Omega)$. Prove that the weak solution $u \in H_0^1(\Omega)$ of the Laplace equation with homogeneous Dirichlet boundary conditions $$-u''=f \text{ in } \Omega, \ ...
user30523's user avatar
  • 1,681
1 vote
0 answers
24 views

Weighted integrability of Sobolev functions in $H^2_p$

Suppose $u \in H^2_p(\mathbb{R}^n)$. What can be said about the decay of $u$ in an $L^p(\mathbb{R}^n)$ sense, i.e. is it true that $$\int_{\mathbb{R}^n} |xu(x)|^pdx < \infty?$$
Rooibos's user avatar
  • 179
0 votes
1 answer
52 views

Convolution of (s+1) characteristic functions

I think it is possible that if I will be able to show that g is in $L^2(\mathbb{R})$ and show that the integral of $|\hat{g}(\omega)|^2(1+\omega)^s d\omega$ over $\mathbb{R}$ is finite, then it is in ...
DDD's user avatar
  • 21
0 votes
1 answer
65 views

permuting two integrations

Can someone explain to me what he used after he said 'and permuting the two integrations with respect t and x we conclude ..' Lemma Let $\Omega$ be an open set of $\mathbb{R}^{N}$ bounded in at least ...
Almendrof66's user avatar
4 votes
1 answer
685 views

An equivalent norm in a subspace of $H^2 (\Omega)$

The following questions concerns a problem I am treating in my Masters dissertation. Let $\Omega $ be an open, bounded domain in $\mathbb{R}^3$. Then the norm $$ \Vert u\Vert^2 = \Vert u\Vert_2^2 + ...
Danilo Gregorin Afonso's user avatar
0 votes
1 answer
63 views

If $\,-u''+u=f$, then $\,\|u\|_{L^s}+\|u'\|_{L^q}+\|u''\|_{L^p}\le c\|f\|_{L^p}$

Let $u\in \mathcal{S}(\mathbb{R})$ (Schwartz space) be a solution of the equation $$ -u''+u=f, \quad \text{where}\,\,\,f \in \mathcal{S}(\mathbb{R}). $$ Show that for all $1\leq p,q,s \leq \...
user30523's user avatar
  • 1,681
0 votes
1 answer
43 views

Weakly equal functions and its decomposition

Suppose $f - g \in V$ where $V$ is a Hilbert space. If $f, g \in V$ and $$ \langle f-g, v\rangle_V = 0, \forall v \in V^*, $$ we say $f$ and $g$ are weakly equal in $V$. I am wondering if there is a ...
induction601's user avatar
  • 1,944
2 votes
2 answers
443 views

Strong $L^2$ convergence to zero implies weak convergence in $H_0^1$?

Suppose $f_k \to 0$ in $L^2(U)$ where $U$ is an open and bounded domain in $\mathbb{R}^n$. Suppose further that $f_k \in H_0^1(U)$ where $H_0^1(U)$ is the Sobolev space whose trace being zeros. I ...
induction601's user avatar
  • 1,944
4 votes
2 answers
713 views

Find a weak differentiable function which is discontinuous

I'm seeking a weak differentiable function $f \in W^{1,1}(\Omega)$ which is discontinuous. I think every weak differentiable function defined on $\mathbb{R}$ is continuous by Sobolev embedding ...
Willy Chen's user avatar
1 vote
1 answer
382 views

Local vs global Sobolev space

Let \begin{equation} H_{\text{loc}}^{2}(\mathbb{R}^d)=\{u:\mathbb{R}^d\to\mathbb{R}\;|\;u\in H^{2}(V)\text{ for all }V\subset\subset \mathbb{R}^d\}. \end{equation} I was wondering if it is true that \...
popoolmica's user avatar
2 votes
1 answer
55 views

Integral of $f(x,y) \in W^{1,1}(B)$

I have the follow problem. Let $B \subset\Bbb R^2 $ the unitary open ball, and let $$ f(x,y):= \begin{cases} 1+x^2+y^2 & \text{ if }x>0,\\ a(y-1)^2 +b(y+x^2) & \text{ if }x<0, \end{...
Alessar's user avatar
  • 500
2 votes
0 answers
132 views

Question about the definition of Bessel potentials $H^{s, p}(\mathbb{R}^n)$ and their embeddings

Usually the Bessel potential spaces are defined as $$ H^{s, p}(\mathbb{R}^n)=\{u\in\mathscr{S}^{\prime}(\mathbb{R}^n):\mathcal{F}^{-1}(\langle\cdot\rangle^s\hat{u})\in L^p(\mathbb{R}^n)\} $$ only for ...
Infinitebig's user avatar
3 votes
1 answer
59 views

If $u_n \rightharpoonup u$ in $H_0^1 (\Omega)$ then $u_n^2 \to u^2$ in $L^{6/5}(\Omega)$

My question regards a minor detail in the proof of a lemma in a research paper I am reading. Let $\Omega$ be an smooth, bounded open set of $\mathbb{R}^3$ and suppose $u_n \rightharpoonup u$ in $H_0^...
Danilo Gregorin Afonso's user avatar
1 vote
1 answer
66 views

Does $\|f\|_{L^2}\,\le\,C(\|f'\|_{L^2} + \|xf\|_{L^2})$ for $f\in H^1(-1,1)$?

I would like to prove that there is a constant $C>0$ such that for all $f\in H^1(-1,1)$ (the usual Sobolev space) we have $$ \|f\|_{L^2}\,\le\,C(\|f'\|_{L^2} + \|xf\|_{L^2}). $$ Here, $xf$ is just ...
amsmath's user avatar
  • 10.6k
3 votes
1 answer
402 views

Trace operator in bounded domain that is not Lipschitz

I know that Trace operator $T : W^{1,p}(\Omega) \to L^p(\partial \Omega, H^{n-1})$ is well defined when $\Omega$ is a bounded Lipschitz domain. (here $H^n$ is the Hausdorff measure). What about only ...
Davide Motta's user avatar
  • 1,068
2 votes
1 answer
191 views

A funny exercise with bounded Lipschitz domain and Sobolev spaces

I'm in trouble with this exercise Let $\Omega \subset \mathbb{R^n}$ be a bounded Lipschitz domain and let $p \in ]1,n[$. prove that there exists $C>0$ such that, for every $f \in W^{1,p}(\Omega)$...
Davide Motta's user avatar
  • 1,068
2 votes
0 answers
268 views

Density of $C^{\infty}(I) \cap W^{1,\infty}(I)$ in $W^{1,\infty}(I)$

I was inspired by this question and Meyers-Serrin theorem to prove that there exists a function $u \in W^{1,\infty}(I) $ such that there are no functions $u_m \in C^{\infty}(I) \cap W^{1,\infty}(I)$ ...
Davide Motta's user avatar
  • 1,068
0 votes
0 answers
332 views

Variational formulation Poisson equation (1d FEM)

I have the same question of this one $\bullet $ In the answer I've seen that a "Lifting" function is used, is it to have a formulation where the test functions are in the same space of the solution? ...
VoB's user avatar
  • 1,603
1 vote
0 answers
114 views

About extension on Sobolev space

Let $\Omega\subset\mathbb{R}^n$ be open connected with smooth boundary. Let $\zeta$ be a zero extension operator : $\forall u\in W^{k,p}(\Omega)$, $$\zeta u := \Big\{\begin{array}uu \quad\text{on $\...
Jingeon An-Lacroix's user avatar
1 vote
0 answers
50 views

Fixed Point Theorem in Topological vector spaces

Suppose we have a decreasing family of Banach spaces $H^s$ (the prototype is some variant of the Sobolev space $H^s=W^{s,2}$, so you can assume that they are Hilbert spaces if you want), and we ...
Qiuye Jia's user avatar
0 votes
1 answer
42 views

A density problem in Sobolev space?

For $ k,p \in \mathbb{N} $, denote $ k $ order classical Sobolev space on $ (0,2\pi) $ by $ H^k(0,2\pi) $ and define \begin{equation*} \mathcal{H}^{k+p}_0:= \{ \varphi \in H^{k+p}(0,2\pi): \varphi(0) =...
Yidong Luo's user avatar
1 vote
1 answer
145 views

Weak limit of gradient of functions

Consider $\{f_n\}\subset H^1(\Omega)$, where $\Omega\subset\mathbb{R}^2$ is an open bounded set. Assume we have $\|\nabla f_n\|_{L^2(\Omega)}\le C$ for a constant $C$ uniformly. Then, we can conclude ...
Syoung's user avatar
  • 670
0 votes
1 answer
68 views

$u \in W^{1,p} (U)$ Implies $u^{+}, u^{-} \in W^{1,p}(U)$ for $U \subseteq \mathbb{R}^N$ Bounded (Evans, Chapter 5, Exercise 18)

Full Problem Assume $1 \leq p \leq \infty$, $U \subseteq \mathbb{R}^N$ bounded. Show that $u \in W^{1,p} (U)$ implies $u^{+}, u^{-} \in W^{1,p}(U)$, and $Du^+ = \begin{cases} Du, \ \text{a.e. on } \...
gbnhgbnhg's user avatar
  • 457
2 votes
0 answers
331 views

Compact Embeddings

Let $\Omega$ be a bounded open with no assumptions on the boundary $\partial\Omega$. Show that $H^{k+1}_0(\Omega)$ is compactly embedded in $W^{k,2}(\Omega)$. Show that if $\Omega$ is bounded, then $...
Jama's user avatar
  • 541
0 votes
1 answer
31 views

Is $\left\{ u\in H^{1}\left(\Omega\right)\left|\,a<u\left(x\right) <b\:\text{a.e.}\right.\right\}$ open in $H^1(\Omega)$

Let $\Omega$ bounded, open, connected and Lip. domain.Let positive constants $a,b$ such that $a<b$. Is $A = \left\{ u\in H^{1}\left(\Omega\right)\left|\,a<u\left(x\right) <b\:\text{a.e.}\...
Joshua's user avatar
  • 85
4 votes
2 answers
108 views

Exact solution $\int_0^1 u'v'=v(1/2)$

This question concerns a variational form of the Laplace equation with homogeneous Dirichlet boundary conditions: $$-u''=f \text{ on } [0,1], u(0)=u(1)=0.$$ Let $V=H^1_0(\Omega), \Omega=[0,1]$ and $...
user30523's user avatar
  • 1,681
1 vote
0 answers
241 views

Generalized definition of the Green Formula (In Sobolev Spaces)

Given the classical version of Green's Formula: Let $\Omega$ be an open bounded subset of $\mathbb{R}^{n}$ and $u, v \in C^{2}(\overline{\Omega})$. \begin{equation} \int_{\Omega} Dv \cdot Du \ dx = -...
Rodrigo's user avatar
  • 73
0 votes
1 answer
191 views

is $H^{1}\left(\Omega\right)\cap L^{\infty}\left(\Omega\right)$ closed in $H^{1}\left(\Omega\right)$?

Let $\Omega$ bounded, open, connected and Lip. domain. Is $H^{1}\left(\Omega\right)\cap L^{\infty}\left(\Omega\right)$ closed in $H^{1}\left(\Omega\right)$? i.e., with the norm of $H^{1}\left(\Omega\...
Joshua's user avatar
  • 85
1 vote
1 answer
87 views

If $\int_U |Du|^2 dx + \int_{\partial U}u^2 dx < \infty$ is $\int_U |u|^2 dx < \infty$?

Do we have $\int_U |Du|^2 dx + \int_{\partial U}u^2 dx < \infty \implies \int_U |u|^2 dx < \infty$ when $u \in H^1(U)$ and $\partial U $ is smooth ? This result is true for $u \in H_0^1(U)$ ...
acreativename's user avatar
0 votes
1 answer
33 views

$u_n\rightarrow u$ and $\nabla u_n\rightarrow g$ implies $g=\nabla u$?

Suppose that $u_n\in H^1(\Omega)$ and $u_n\rightarrow u$ in $L^2(\Omega)$ and one shows that $\nabla u_n$ has a limit in $L^2(\Omega)$, let's say $\nabla u_n\rightarrow g$. I wonder if $g=\nabla u$. ...
Senna's user avatar
  • 1,233
4 votes
0 answers
77 views

How to make sense of Sobolev spaces on bounded domain from the point of view of distributions.

In the theory of $\mathbb{R}^{d}$, one defines the space of Schwartz functions $ \mathcal{S}(\mathbb{R}^{d}) $ to be the space of smooth functions decaying faster than any polynomials. However, when ...
Meagain's user avatar
  • 693
0 votes
0 answers
238 views

A little help to show strong convergence in Sobolev space $H^1$

Let $u_n $ be a sequence in $H^1 (\Omega,\mathbb{C})$ where $\Omega \subset \mathbb{R} ^N$ is bounded. Assume that we know that all the functions $u_n$ are smooth and $\Vert u_n \Vert _{C^m} \leq K(m,...
Senna's user avatar
  • 1,233

1
37 38
39
40 41
111